Anuncio

Colapsar
No hay ningún anuncio todavía.

Duda sobre láseres e interferencias destructivas

Colapsar
X
 
  • Filtro
  • Hora
  • Mostrar
Borrar todo
nuevos mensajes

  • 1r ciclo Duda sobre láseres e interferencias destructivas

    Buenas, desde hace ya unos meses que tengo una duda relacionada con los láseres y las interferencias destructivas. Quería saber si me podéis sacar de dudas.

    Pues bien, se sabe que en una interferencia de ondas, existen puntos donde la amplitud es nula (interferencia destructiva) y donde se maximiza (interferencia constructiva), como se puede ver en el experimento de la doble rendija.
    Mi duda es que, en caso de tener 2 láseres en contra-fase, donde sólo se produzca interferencia destructiva, ¿a dónde va la energía? ¿la onda viaja con amplitud 0 aunque no se vea? ¿o simplemente cambia de dirección?
    Digamos que no sé si la onda desaparece o cambia de dirección (como he leído en alguna parte).

    Gracias por las respuestas.


    Saludos.

    PD: He estado viendo varios posts antes de hacer la pregunta, pero en ningún caso he visto que se resuelva mi duda de qué pasa con el láser.

  • #2
    Re: Duda sobre láseres e interferencias destructivas

    Hola bienvenido al foro, como nuevo miembro te sera útil leer Consejos para conseguir- ayuda de forma efectiva
    La suma de las energías de cada onda, no es igual a la energía de la onda resultante, puedes verlo en que la interferencia constructiva de dos ondas en fase crea una onda que cuadriplica el valor energético de las ondas individuales.
    http://laplace.us.es/wiki/index.php/...ción_de_ondas

    Comentario


    • #3
      Re: Duda sobre láseres e interferencias destructivas

      Gracias por el enlace, me ha aclarado bastantes cosas.
      Entonces, si ambas ondas tienen el mismo sentido y están en contrafase, básicamente desaparecerían, ¿no? ¿pero a dónde va a parar su energía? Lo que yo entiendo es que se anulan entre sí.

      Y en caso de que pusiéramos un láser enfrente de otro (teniendo misma amplitud y frecuencia), el láser resultante sería parpadeante? iría "encendiéndose" y "apagándose"? No se si me explico.

      Comentario


      • #4
        Re: Duda sobre láseres e interferencias destructivas

        Escrito por XnetLoL Ver mensaje
        ¿pero a dónde va a parar su energía? Lo que yo entiendo es que se anulan entre sí.
        Es una pregunta que se contesta mas fácil desde el punto de vista cuántico, que tengo poca idea ,pero te diré lo que entiendo y si me equivoco que alguien me corrija.

        En todo momento la onda tiene una energía mecánica . una parte es energía potencial y otra cinetica, intuyo que al sumarse destructivamente la componente cinetica se anula y la potencial se hace máxima.




        Escrito por XnetLoL Ver mensaje
        Y en caso de que pusiéramos un láser enfrente de otro (teniendo misma amplitud y frecuencia), el láser resultante sería parpadeante? iría "encendiéndose" y "apagándose"? No se si me explico.
        claro es el principio usado para la holografía sobre líquidos , gases o humos.

        http://hyperphysics.phy-astr.gsu.edu...mod/holog.html

        Comentario


        • #5
          Re: Duda sobre láseres e interferencias destructivas

          Escrito por XnetLoL Ver mensaje
          ... Mi duda es que, en caso de tener 2 láseres en contra-fase, donde sólo se produzca interferencia destructiva, ¿a dónde va la energía? ...
          Escrito por XnetLoL Ver mensaje
          ... Entonces, si ambas ondas tienen el mismo sentido y están en contrafase, básicamente desaparecerían, ¿no?

          Hola XnetLoL, observa que lo que planteas no es realizable físicamente. Un láser situado en el origen puede generar la onda de campo eléctrico roja o la onda azul pero no ambas a la vez. Para generar 2 ondas distintas has de tener 2 láser, pero entonces es imposible que estén en el mismo punto físico, por lo tanto es imposible que tengas la pareja de ondas exactamente compensadas en todos los puntos del espacio que hay en el dibujo.

          Si tienes 2 láser, cada uno genera en cada punto del espacio un campo eléctrico y un campo magnético que son función del tiempo. El teorema de superposición te permite afirmar que el campo eléctrico en cualquier punto será la suma vectorial del campo eléctrico generado por el primer láser más el campo eléctrico generado por el segundo láser, en total un campo de amplitud e(t). Y es imposible que e(t)=0 en todos los puntos del espacio si no apagas los 2 láser. Ello significa que la densidad volumétrica de energía eléctrica en un punto es:



          Del mismo modo, el teorema de superposición te permite afirmar que el campo magnético en cualquier punto será la suma vectorial del campo magnético generado por el primer láser más el campo magnético generado por el segundo láser, en total un campo de amplitud b(t). Ello significa que la densidad volumétrica de energía magnética en ese punto es:



          La densidad de energía volumétrica total será:



          Para calcular la energía total U en un determinado volumen V, hay que integrar esa expresión en el volumen deseado:



          Supongo que puedes conseguir (por ejemplo mediante espejos como dice Richard en #6) ciertos volúmenes de espacio en los que y pero no puedes conseguir que sea cero en todos los puntos del espacio, a no ser que no haya ningún láser encendido.

          Saludos.
          Última edición por Alriga; 21/01/2019, 16:46:55. Motivo: Añadir cita de XnetLoL de la pregunta que intento responder
          "Das ist nicht nur nicht richtig, es ist nicht einmal falsch! "

          Comentario


          • #6
            Re: Duda sobre láseres e interferencias destructivas

            Escrito por Alriga Ver mensaje
            Hola XnetLoL, observa que lo que planteas no es realizable físicamente
            Eso pensé al principio, luego me acorde del espejo de salida de los interferómetros

            Escrito por Richard R Richard Ver mensaje
            intuyo que al sumarse destructivamente la componente cinetica se anula y la potencial se hace máxima
            Esto era para ondas mecánicas ,no electromagnéticas?

            Comentario


            • #7
              Re: Duda sobre láseres e interferencias destructivas

              Efectivamente las fuentes deben ser independientes, debido a que la característica de la señal es la coherencia y dado que la señal EM es descrita como:



              Donde el vector A es el vector potencial magnético y es posible obtener los valores del campo E y B a partir de este.

              En la situación planteada los vectores son opuestos:



              De aquí se desprende que si la fuente no es independiente, los valores del campo son:



              Lo mismo para el campo magnético

              Y por lo tanto la fuente no emite radiación, por lo tanto no aporta energía al campo.De esta manera la conservación de la energía permanece.

              Para el caso que menciona Alriga, donde existen fuentes independientes, la interferencia destructiva solo ocurre en un volumen fijo del espacio, donde los frentes de onda convergen con misma amplitud y una diferencia de fase de 90º. ¿Que ocurre con la energía del campo electromagnético?
              Pues la energía, que es proporcional a la amplitud es cero en dicho volumen. Al igual que el momento de la onda.

              Dado el volumen V tal que integra en su centro a las fuentes equidistantes y al punto de interferencia destructiva y Pf es la potencia de ambas fuentes independientes, donde el tiempo T abarca el tiempo desde que parten las ondas hasta el punto de interferencia destructiva:



              Por lo tanto debido a la diferencia de caminos entre las 2 ondas + la apertura de los frentes de onda y su ancho del beam inicial, eso implica necesariamente que deba darse la interferencia es más de un punto del espacio.
              Solo sucedería en un punto si la apertura de la radiación (Apertura numérica del haz) sería 0, y su ganancia respecto a antena isotrópica sería infinito, aspecto no posible físicamente.

              El mínimo ancho de laser debe ser mayor a la longitud de onda, debido a que:



              Si el ancho es igual a la longitud de onda, el haz inicial tendría una divergencia máxima. Así, si se quisiera que la apertura sea 1 mrad, la relación de ancho de beam inicial y lambda debe ser de 1000 veces.

              Aún así, dado ese dato de color, en el voxel donde se da la interferencia puede darse una mayor contribución de interferencia destructiva.

              ¿Porqué no se mantiene la conservación de la energía si consideramos el instante de tiempo desde que las fuentes comienzan a transmitir hasta que los frentes de ondas arriban al voxel de interferencia?
              Pues yo creo que no se cumple la simetría temporal, es decir, la densidad langrangiana del campo electromagnético es dependiente del tiempo y del espacio.
              ¿Qué me dicen?
              Por más bella o elegante que sea la teoría, si los resultados no la acompañan, está mal.

              Comentario


              • #8
                Re: Duda sobre láseres e interferencias destructivas

                Gracias a todos por las respuestas. En efecto, yo también creía que era posible que dos ondas con la misma amplitud, frecuencia y sentido se solaparan (con algo de óptica, como bien ha indicado Richard R Richard). De todos modos se me hace extraño que dos ondas electromagnéticas sigan propagándose una vez se produce una interferencia destructiva entre ellas (es decir, en el instante en el que dos ondas con sentido diferente se anulan), ya que si en un instante no hay ni campo magnético ni eléctrico, no entiendo como puede generarse otro siguiendo con el ciclo de la onda.

                En cuanto a tu última cuestión (Julián) sobre la simetría temporal, no podría responderte, esos conocimientos me superan (hago física en la uni pero mayoritariamente la aprendo por mi cuenta).

                Seguiré investigando, un tema muy interesante, desde luego.

                Saludos.
                Última edición por XnetLoL; 22/01/2019, 01:10:48.

                Comentario


                • #9
                  Re: Duda sobre láseres e interferencias destructivas

                  Escrito por Julián Ver mensaje

                  ¿Porqué no se mantiene la conservación de la energía si consideramos el instante de tiempo desde que las fuentes comienzan a transmitir hasta que los frentes de ondas arriban al voxel de interferencia?
                  Pues yo creo que no se cumple la simetría temporal, es decir, la densidad langrangiana del campo electromagnético es dependiente del tiempo y del espacio.
                  ¿Qué me dicen?
                  Hola. Dos pulsos de ondas, localizados y separados inicialmente, pueden interferir cuando los pulsos llegan a solapar. La energía total del campo electromagnético será siempre igual a la suma de las energias iniciales, aunque la manera en la que se distribuye esa energia espacialmente dependerá de la interferencia. En algunos puntos será destructiva, y en otros será cosntructiva.

                  Lo que no ocurre, en ningun instante de tiempo, es que en todo el espacio se dé interfencia destructiva. En unos puntos se da interferencia constructiva, y en otros interferencia destructiva. Al fin y al cabo, esto es el experimento de la doble rendija (con ondas electromagnéticas, nada de cuántica).

                  Una imagen visual (con electrones, pero sería lo mismo con campo electromagnético) https://en.wikipedia.org/wiki/Electr...t_interference.

                  Un saludo

                  Comentario


                  • #10
                    Re: Duda sobre láseres e interferencias destructivas

                    gracias carroza.

                    Si, anteriormente había intentado buscar la relación de interferencia destructiva-constructiva pero eso dependerá del ancho del beam de los laser. Como este es mayor a la longitud de onda, necesariamente, se deben dar puntos con interferencia constructiva-destructiva. Esto se deriva de la divergencia.

                    Consideremos 2 láseres iguales pero con la condición de que llegan a un punto medio donde interfieren destructivamente (o constructivamente; dándose esa diferencia de fase debido a las fuentes o diferencia de caminos, eso no es importante) se da, debido a las secciones de los mismos, contribuciones constructivas y destructivas. La siguiente es una imagen a modo de ejemplo donde vemos una relación de 1 longitud de onda (solo a modo de ejemplo para planteo ya que debido a la difracción esa relación no es posible para tener una dispersión de haz pequeña).

                    Haz clic en la imagen para ampliar

Nombre:	Untitled.jpg
Vitas:	1
Tamaño:	19,2 KB
ID:	304396

                    Donde el ancho del lóbulo principal es y D es la distancia desde la fuente hasta el punto de interferencia y a el ancho de la rendija del láser. Esta relación se desprende de la difracción del haz en la apertura de la "óptica del laser"

                    Considerando que la dispersión de los hazes es mínima (apertura exagerada en la imagen), para que halla la misma contribución de interferencia destructiva que constructiva se debe dar que el ancho del beam debe ser una relación entera a la longitud de onda :

                    Es decir, si entonces donde n es un valor entero.

                    Y viceversa por lo que

                    ¿Qué sucede si la relación no es entera? ¿Qué parámetro impone que la relación siempre es entera?

                    Se que algo debe estar faltando en mi razonamiento, ya que en caso contrario, echamos por la borda la conservación de energía.

                    Así que para que esto no suceda, debo comenzar planteando que será la misma contribución constructiva y destructiva, para que se conserve la energía y de allí probarlo.
                    Para cumplir con este requerimiento, hasta ahora lo único que puedo decir es que el volumen de convergencia de los beams debe tener 2 lados transversales paralelos a la dirección de desplazamiento de las ondas de longitud donde . Es decir, una relación entera a la longitud de onda, evidentemente estoy dejando de lado el aumento de la sección de los hacez (divergencia del haz) a lo largo de su trayectoria en este análisis (lo que lo complicaría) y solo centrándome en la cintura del haz ("ancho más pequeño" en el punto de partida de las fuentes).

                    Podemos ver el espectro de difracción de rendija simple acá http://hyperphysics.phy-astr.gsu.edu...t/mulslid.html

                    https://luz.izt.uam.mx/wiki/index.php/Ondas:_Gaussianas
                    Última edición por Julián; 23/01/2019, 18:01:41.
                    Por más bella o elegante que sea la teoría, si los resultados no la acompañan, está mal.

                    Comentario


                    • #11
                      Re: Duda sobre láseres e interferencias destructivas

                      Escrito por carroza Ver mensaje

                      Lo que no ocurre, en ningún instante de tiempo, es que en todo el espacio se dé interferencia destructiva.
                      En el ejemplo de la suma de ondas refractada y reflejadas a la salida el interferómetro, porque , me pregunto, en el espacio de azul (del dibujo inferior) la superposición no es destructiva? no solo en un instante sino en todos los instantes...supongamos condiciones ideales de igualdad de amplitudes dispersión ,ancho del rayo y longitudes de onda coherentes... no se algo de me escapa, entiendo que la energía no se puede perder, pero entonces en que parte del espacio esta la parte constructiva?



                      Escrito por Julián Ver mensaje
                      [ATTACH=CONFIG]14060[/ATTACH]
                      Creo que en tu dibujo las ondas no recorren el mismo camino óptico como creo que se había propuesto

                      Escrito por XnetLoL Ver mensaje
                      Entonces, si ambas ondas tienen el mismo sentido y están en contrafase,
                      a la salida del interferómetro tememos algo así,la onda verde se refleja y cambia de fase, si el camino óptico al atravesar el espesor del espejo semitransparente, es proporcional a la longitud de onda, cuando la onda roja llega en fase al espejo se sumará destructivamente.... de ahí entonces su pregunta erá adónde fue la energía de la onda roja refractada y de la verde reflejada?

                      Haz clic en la imagen para ampliar

Nombre:	interf.png
Vitas:	1
Tamaño:	7,6 KB
ID:	304397
                      o que es lo que me pierdo algo de la explicación?

                      Comentario


                      • #12
                        Re: Duda sobre láseres e interferencias destructivas

                        A ver, si tenemos un pulso localizado de radiación, no puede haber una unica longitud de onda. Un pulso localizado es un paquete de ondas, en el que hay controbucion de todas las longitudes de onda, en un intervalo determinado por el tamaño del paquete. Cuanto más pequeño sea el paquete, más grande es el intervalo de longitudes de onda que necesitamos para formarlo.

                        Por otro lado, en la radiación electromagnética, como en cualquier tipo de ondas, la conservación de la energía viene dada por una ecuación de continuidad. la variación en el tiempo de la energia integrada en un volumen determinado es igual al flujo de energía (el vector de Poynting) a través de la superficie que limita ese volumen. Si consideramos un volumen grande, en cuya superficie no haya radiación electromagnética de ningun tipo, el vector de poynting se anula en la superficie, con lo cual la energia en el interior no varía con el tiempo. No tenemos que ir buscando "a mano" cancelación entre interferencias constructivas y destructivas, ya que las propia ecuación de continuidad nos asegura la conservación de la energía.

                        Saludos

                        Saludos

                        Comentario


                        • #13
                          Re: Duda sobre láseres e interferencias destructivas

                          Realicé una simulación gráfica y numérica; y obtengo que la conservación de la energía se mantiene (valla novedad pero a mi favor esto no siempre sucede, si no se cumplen las simetrías temporales como en el caso de la expansión del espacio-tiempo). Yo en cambio, en mi cabeza, sigo sin poder ver la conservación. Sé que debe haber algo mal en mi razonamiento y ahora ya confirmado visualmente.

                          Por lo tanto hay una simetría entre las contribuciones destructivas y contructivas siempre (misma cantidad de minimos que máximos), sin importar la forma y el ángulo de incidencia de los hacez (aunque me falta simular cuando ambos beams tienen diferente dispersión, apertura), lo que no puedo es encontrar la relación matemática que lo sustente. ¿como es posible esto independientemente de la geometría de la intersección?

                          PD: llevo un tiempo sin programar

                          Haciendo una simulación en python, con dos haces laser coherentes sin dispersión de los mismos obtengo:

                          Sean las señales (tomando una instantea en el tiempo):

                          y

                          Longitud de onda igual a 100 pixeles.

                          Haz clic en la imagen para ampliar

Nombre:	intersección.png
Vitas:	1
Tamaño:	31,9 KB
ID:	304399

                          Ahora si desfaso uno del otro

                          y

                          obtengo:

                          Haz clic en la imagen para ampliar

Nombre:	intersección2.png
Vitas:	1
Tamaño:	31,7 KB
ID:	304400

                          Si ahora no tomamos una instantánea, sino diferentes instantes de tiempo podemos ver la simulación completa

                          Haz clic en la imagen para ampliar

Nombre:	2rxve3.gif
Vitas:	1
Tamaño:	252,1 KB
ID:	304404

                          Si sumo el cuadrado de los valores de cada beam por separado (sin que interfieran)y lo sumo ( ) (integro) obtengo: 500000

                          Ahora si realizo el cuadrado de la suma () (es decir, considerando la interferencia) obtengo: 499999.99999995285

                          ('valor densidad de energia beam 1: ', 250000.0)
                          ('valor densidad de energia beam 2: ', 250000.00000000108)
                          ('suma del cuadrado de ambos valores por separado: ', 500000.00000000105)
                          ---------
                          ('Cuadrado de la suma de ambos valores: ', 499999.99999995285)


                          hay una pequeña diferencia. Es decir practicamente nula debido a la discretización de los valores en la simulación.

                          Ya que si las señales tienen la forma:

                          y , cambiando la longitud de onda a 10 pixeles

                          Tenemos pues los siguientes valores:

                          Haz clic en la imagen para ampliar

Nombre:	intersección3.png
Vitas:	1
Tamaño:	10,8 KB
ID:	304398

                          ('valor densidad de energia beam 1: ', 249999.99999999997)
                          ('valor densidad de energia beam 2: ', 250000.0)
                          ('suma del cuadrado de ambos valores por separado: ', 500000.0)
                          ---------
                          ('Cuadrado de la suma de ambos valores: ', 500000.0)
                          Es decir practicamente nula y se conserva la energía


                          En el caso anterior la longitud de onda es 100 pixeles (o 10 pixeles) y el ancho de los beams es 500 pixeles por lo que la relación ancho sobre longitud de onda es 5.

                          Si ahora el ancho del beam es 293 pixeles, dando una relación de 2.93, entonces obtengo:

                          Haz clic en la imagen para ampliar

Nombre:	intersección5.png
Vitas:	1
Tamaño:	19,9 KB
ID:	304401

                          ('valor densidad de energia beam 1: ', 250000.0)
                          ('valor densidad de energia beam 2: ', 146499.99999999886)
                          ('suma del cuadrado de ambos valores por separado: ', 396499.99999999884)
                          ---------
                          ('Cuadrado de la suma de ambos valores: ', 396499.99999997375)
                          Pues si, veo que se sigue manteniendo la simetría. Y cambiando una relación a no entera no modifica en nada.

                          ¿Qué pasa si la intersección es a 45º?

                          Haz clic en la imagen para ampliar

Nombre:	intersección8.png
Vitas:	1
Tamaño:	52,1 KB
ID:	304403

                          Haz clic en la imagen para ampliar

Nombre:	intersección7.png
Vitas:	1
Tamaño:	49,7 KB
ID:	304402


                          ('valor densidad de energia beam 1: ', 299999.99999999994)
                          ('valor densidad de energia beam 2: ', 353894.01235959464)
                          ('suma de ambos valores por separado: ', 653894.0123595946)
                          ---------
                          ('Cuadrado de la suma de ambos valores: ', 653890.4055945525)
                          Y acá casi obtenemos lo mismo. Y considerando la diferencia por la discretación, lo tomo como lo mismo. Probablemente realizando una simulación mas optimizada llevando los valores discretos al mínimo sería más claro. Pero es evidente que es lo mismo. Aunque no estoy 100% seguro.

                          A ver, si tenemos un pulso localizado de radiación, no puede haber una unica longitud de onda. Un pulso localizado es un paquete de ondas, en el que hay controbucion de todas las longitudes de onda, en un intervalo determinado por el tamaño del paquete. Cuanto más pequeño sea el paquete, más grande es el intervalo de longitudes de onda que necesitamos para formarlo.
                          Pero en cuanto a la radiación obtenida de un laser podemos considerar idealmente un pulso monocromático. Con una distribución gaussiana con valor medio en la longitud de onda característica del sistema.


                          Código:
                          import numpy as np
                          from scipy import ndimage 
                          import math
                          
                          
                          import matplotlib.pyplot as plt
                          %matplotlib inline
                          
                          
                          '''
                          /////////////////////////////////////////////////////////////////////////////////////////
                          Senal laser 1
                          '''
                          senal_1 = np.zeros((1000,1000))
                          x_1 = np.arange(0,1000,1)
                          
                          
                          
                          
                          #longitud de onda 100 pixeles
                          E_1 = np.cos( (2*math.pi/100)*x_1)
                          
                          
                          
                          
                          #ancho del beam (00-250,500+250), es decir 500
                          for i in range (500-250,500+250,1):
                              senal_1 [i]=(E_1)
                          '''
                          Fin señal laser 1
                          ////////////////////////////////////////////////////////////////////////////////////////
                          Comienzo señal laser 2
                          ''' 
                          aux_senal_2 = np.zeros((1000,1000)) 
                          senal_2 = np.zeros((1000,1000))
                          
                          
                          
                          
                          x_2 = np.arange(0,1000,1)
                          
                          
                          
                          
                          #longitud de onda 100 pixeles
                          E_2 = -np.cos( (2*math.pi/100)*x_2 )
                          
                          
                          
                          
                          #ancho del beam (00-250,500+250), es decir 500
                          for i in range (500-250,500+250):
                              aux_senal_2 [i]=(E_2) 
                          
                          
                          # rotar señal 2 90º para obtener una dirección perperdicular entre beams
                          senal_2=ndimage.rotate(aux_senal_2, 90, reshape=False) 
                          
                          
                          
                          
                          '''
                          Fin señal laser 2
                          ////////////////////////////////////////////////////////////////////////////////////////////
                          '''
                          
                          
                          fig, axes = plt.subplots()
                          axes.imshow(senal_1+senal_2,aspect='auto')
                          plt.show()
                          
                          
                          
                          
                          sum1=0
                          sum2=0
                          sum3=0
                          
                          
                          
                          
                          for i in range (0,1000): 
                              for j in range (0,1000): 
                                  sum1=sum1+np.power(senal_1[i][j],2)
                          
                          
                          for i in range (0,1000): 
                              for j in range (0,1000): 
                                  sum2=sum2+np.power(senal_2[i][j],2)
                          
                          
                          
                          
                          for i in range (0,1000): 
                              for j in range (0,1000): 
                                  sum3=sum3+np.power(senal_1[i][j]+senal_2[i][j],2)
                          
                          
                          print ("valor densidad de energia beam 1: ",sum1)
                          print ("valor densidad de energia beam 2: ",sum2)
                          print ("suma del cuadrado de ambos valores por separado: ",sum1+sum2)
                          print ("---------")
                          
                          
                          
                          
                          print ("Cuadrado de la suma de ambos valores: ",sum3)
                          Última edición por Julián; 24/01/2019, 18:43:58.
                          Por más bella o elegante que sea la teoría, si los resultados no la acompañan, está mal.

                          Comentario


                          • #14
                            Re: Duda sobre láseres e interferencias destructivas

                            Escrito por Julián Ver mensaje

                            Sean las señales (tomando una instantea en el tiempo):

                            y

                            Longitud de onda igual a 100 pixeles.

                            [ATTACH=CONFIG]14066[/ATTACH]
                            Hola. Debes ser consciente de que, si tus campos se cortan bruscamente y van a cero para x o y = 250 y 750, como parece indicar tu figura, ya tus campos no cumplen las ecuaciones de maxwell en todo el espacio, con lo que ya no derivan de un lagrangiano, con lo que ya no tienen que cumplir la conservacion de la energia.

                            Si quisieras hacer una representación realista del campo eletromagnético, tendrias que tomar paquetes de ondas en las direcciones x e y, con lo que los campos se irían a cero lateralmente de forma gradual, nunca brusca. Ahí sí tendrías una conservación estricta de la energía, integrada en todo el espacio, no sólo en el cuadrito que seleccionas para ver la interferencia.

                            Un saludo

                            Comentario

                            Contenido relacionado

                            Colapsar

                            Trabajando...
                            X